0 Daumen
373 Aufrufe

Aufgabe:

Beweis für Summe alternierender Binomialkoeffizienten


Problem/Ansatz:

\(\sum\limits_{k=0}^{n}{\frac{(-1)^{k}}{k+1} }\)\(\begin{pmatrix} n \\ k \end{pmatrix} \)= \( \frac{1}{n+1} \)


Ich habe es mit Induktion versucht, komme dabei nicht weiter, weil ich nicht weiß, wie ich
\(\sum\limits_{k=0}^{n}{\frac{(-1)^{k}}{k+1} }\)\(\begin{pmatrix} n \\ k-1 \end{pmatrix} \)

umformen soll.

Avatar von

1 Antwort

0 Daumen
 
Beste Antwort

Aloha :)

Wir wenden zuerst die Regel \(\binom{n+1}{k+1}=\frac{n+1}{k+1}\binom{n}{k}\) für den Binomialkoeffizienten an:$$\phantom{=}\sum\limits_{k=0}^n\binom{n}{k}\frac{(-1)^k}{k+1}=\frac{1}{n+1}\sum\limits_{k=0}^n\binom{n}{k}\frac{n+1}{k+1}(-1)^k=\frac{1}{n+1}\sum\limits_{k=0}^n\binom{n+1}{k+1}(-1)^k$$

Jetzt machen wir eine Indexverschiebung, indem wir \(k\) von \(1\) bis \(n+1\) laufen lassen und dafür bei den Summanden \(k\) um \(1\) vermindern:$$=\frac{1}{n+1}\sum\limits_{k=1}^{n+1}\binom{n+1}{k}(-1)^{k-1}$$

Würde die Summe bei \(k=0\) beginnen, wäre sie um \(\binom{n+1}{0}(-1)^{0-1}\) größer. Das nutzen wir aus und lassen die Summe tatsächlich bei \(k=0\) beginnen und ziehen den obigen Wert wieder ab.$$=\frac{1}{n+1}\left(\sum\limits_{k=0}^{n+1}\binom{n+1}{k}(-1)^{k-1}\,\underbrace{-\binom{n+1}{0}(-1)^{0-1}}_{=+1}\right)$$

Jetzt kommt der binomische Lehrsatz zur Anwendung:$$=\frac{1}{n+1}\left(-\underbrace{\sum\limits_{k=0}^{n+1}\binom{n+1}{k}\cdot1^{(n+1)-k}\cdot(-1)^k}_{=\text{binomischer Lehrsatz}\implies(1+(-1))^{n+1}}+1\right)$$$$=\frac{1}{n+1}\left(-\underbrace{(1+(-1))^{n+1}}_{=0}+1\right)=\frac{1}{n+1}$$

Avatar von 148 k 🚀

Super schnelle Antwort mit wirklich guter und verständlicher Erklärung. Vielen Dank!

Ein anderes Problem?

Stell deine Frage

Willkommen bei der Mathelounge! Stell deine Frage einfach und kostenlos

x
Made by a lovely community